Introduksjon (VGS): Derivasjon under integraltegnet

Her kan brukere av forum utfordre hverandre med morsomme oppgaver og nøtter man ønsker å dele med andre. Dette er altså ikke et sted for desperate skrik om hjelp, de kan man poste i de andre forumene, men et sted for problemløsing på tvers av trinn og fag.

Moderatorer: Vektormannen, espen180, Aleks855, Solar Plexsus, Gustav, Nebuchadnezzar, Janhaa

Svar
daofeishi
Tyrann
Tyrann
Innlegg: 1486
Registrert: 13/06-2006 02:00
Sted: Cambridge, Massachusetts, USA

Her skal jeg introdusere en teknikk som ikke ofte læres bort i introduksjonstekster i integrasjons-/derivasjonsregning, men som kan komme til nytte i flere sammenhenger. Teknikken heter integrasjon under derivasjonstegnet, og den rimelig dyktige fysikeren Richard Feynman er kjent for å ha sagt:
Richard Feynman skrev:I used that one damn tool again and again
Denne morsomme saken fungerer som følger. Tenk deg at du har en funksjon av to variable [tex]f(x,n)[/tex]. (Der både x og n er reelle tall.)

En slik funksjon kan for eksempel se slik ut:
[tex]f(x,n) = \frac{1}{x^2+n^2}\\f(x,n) = \frac{x}{n}+n \sin(x)[/tex]

Vi dypper nå foten i et emne som kalles "multivariabel kalkulus" - teorien for integrasjon/derivasjon av funksjoner med mer enn én variabel. Har vi kjennskap til regning med en variabel, blir ikke overgangen et problem.

Du kjenner antakeligvis til notasjonen [tex]\frac{\rm{d}}{\rm{d}x} f(x)[/tex] for et derivat i énvariabel kalkulus. [tex]f(x, n)[/tex] har 2 variabler, og kan dermed deriveres med respekt til 2 ulike variabler - x eller n. Disse noteres slik:
[tex]\frac{\partial}{\partial x}f(x, n), \qquad \frac{\partial}{\partial n}f(x, n)[/tex]
Når vi deriverer med respekt til en variabel, later vi som alle andre variabler er konstanter. Her følger et eksempel:

[tex]f(x, n) = n \sin(x)[/tex]
Da er
[tex]\frac{\partial}{\partial x}f(x, n) = n \cos(x) \\ \frac{\partial}{\partial n}f(x, n) = \sin(x)[/tex]
Hvorfor? Fordi i siste derivatet er [tex]\sin(x)[/tex] bare som en konstant å regne(!), og n variabelen vi deriverer med hensyn på

Et interessant resultat er at dersom du har et definitt integral med respekt til ene variabelen, har du lov til å derivere andre UNDER INTEGRALTEGNET om du vil - la oss ta et enkelt illustrativt eksempel først:

Som vi vet er
[tex]\int _1 ^a \frac{1}{nx} \rm{d}x = \frac{\ln(a)}{n}[/tex]
Vi har så lov til å derivere under integraltegnet:
[tex]\int _1 ^a \frac{\partial}{\partial n} \frac{1}{nx} \rm{d}x = \frac{\partial}{\partial n}\frac{\ln(a)}{n} \\ \int _1 ^a - \frac{1}{n^2x} \rm{d}x = -\frac{\ln(a)}{n^2}[/tex]

Dette så vel kanskje vel og greit ut - dette var jo rimelig trivielt. Men denne teknikken kan har sine store styrker... La oss si du ønsker å finne ut hva følgende integral er, for alle positive heltall k:
[tex]\int _{-\infty}^\infty \frac{1}{(x^2+1)^k} \rm{d}x[/tex]

Du vet allerede/får oppgitt at [tex]\int \frac{1}{x^2 + 1}\rm{d}x = \arctan(x) + C[/tex]

Dette kan synes som et vanskelig problem, helt til vi innfører en liten hjelpevariabel n. Skriv om problemet slik:
[tex]\int _{-\infty}^\infty \frac{1}{(x^2+n)^k} \rm{d}x[/tex]
Vi vet selvsagt at n = 1, men dette bruker vi senere.

La oss begynne med k = 1:
[tex]\int _{-\infty}^\infty \frac{1}{x^2+n} \rm{d}x = \left[ \frac{1}{\sqrt n} \arctan ( \frac{x}{\sqrt n} ) \right] _{-\infty} ^\infty = \frac{\pi}{\sqrt n}[/tex]

(Vi ser her ved å sette n=1 at [tex]\int _{-\infty}^\infty \frac{1}{x^2+1} \rm{d}x = \pi[/tex])

Hva skjer så dersom vi deriverer under integraltegnet med respekt på n?
[tex]\int _{-\infty}^\infty \frac{1}{x^2+n} \rm{d}x = \frac{\pi}{\sqrt n} \\ \int _{-\infty}^\infty \frac{\partial}{\partial n} \frac{1}{x^2+n} \rm{d}x = \frac{\partial}{\partial n} \frac{\pi}{\sqrt n} \\ \int _{-\infty}^\infty - \frac{1}{(x^2+n)^2} \rm{d}x = -\frac{\pi}{2n^{\frac 3 2}} \\ \int _{-\infty}^\infty \frac{1}{(x^2+n)^2} \rm{d}x = \frac{\pi}{2n^{\frac 3 2}} [/tex]

Og når n = 1, ser vi at:
[tex]\int _{-\infty}^\infty \frac{1}{(x^2+1)^2} \rm{d}x = \frac{\pi}{2}[/tex]


Første oppgave lyder som følger: Undersøk videre. Fortsett å derivere under integraltegnet. Kan du finne et mønster for [tex]\int _{-\infty}^\infty \frac{1}{(x^2+1)^k} \rm{d}x[/tex]? Prøv å finne fram til en generell formel om du klarer!
sEirik
Guru
Guru
Innlegg: 1551
Registrert: 12/06-2006 21:30
Sted: Oslo

daofeishi skrev: ...

[tex]\int _{-\infty}^\infty \frac{1}{(x^2+n)^2} \rm{d}x = \frac{\pi}{2n^{\frac 3 2}} [/tex]
*Dra frem gamle, støvete derivasjonsregler*

[tex]u = x^2 + n[/tex], [tex]\frac{\partial}{\partial n} u = 1[/tex]

[tex]\frac{\partial}{\partial n}\ \frac{1}{(x^2+n)^2} = \frac{\partial}{\partial n} u^{-2} = -2u^{-3} = -2\frac{1}{(x^2+n)^3}[/tex]

[tex]\frac{\partial}{\partial n}\ \frac{\pi}{2}n^{-3/2} = -\frac{3\pi}{4}n^{-5/2}[/tex]

[tex]\int _{-\infty}^\infty -2\frac{1}{(x^2+n)^3} \rm{d}x = -\frac{3\pi}{4}n^{-5/2} [/tex]

[tex]\int _{-\infty}^\infty \frac{1}{(x^2+n)^3} \rm{d}x = \frac{3\pi}{8}n^{-5/2} [/tex]

Og ved å sette [tex]n=1[/tex] ser vi at [tex]I_3 = \frac{3\pi}{8}[/tex].

"Jukser litt." Beregner [tex]I_4[/tex] tilnærmet, numerisk på kalkulator, og tipper hva det blir nøyaktig.

[tex]I_4 = \frac{25}{8}\pi[/tex]

Lengre enn det tør jeg ikke å stole på det kalkulatoren sier, så med mindre noen har funnet et mønster her, så er det vel bare å sette seg ned og derivere litt til! Eller kanskje derivere induktivt og se om en får en rekke [tex]a_n[/tex] som gir tallene som skal være på høyresiden.
Janhaa
Boltzmann
Boltzmann
Innlegg: 8552
Registrert: 21/08-2006 03:46
Sted: Grenland

Jeg deriverte litt videre, der Eirik slapp. Ikke at mønsteret åpenbarte seg av den grunn. Forbehold, har slurva mye i det siste :!:


[tex]I_1=\pi[/tex]

[tex]I_2={\pi \over 2}[/tex]

[tex]I_3={3\pi \over 8}[/tex]

[tex]I_4={5\pi \over 16[/tex]

[tex]I_5=\frac{35\pi}{128}[/tex]

[tex]I_6=\frac{63\pi}{256}[/tex]

[tex]I_7=\frac{189\pi}{1024}[/tex]

Orker ikke mer nå, oddetals pi i teller og 2[sup]p[/sup] i nevner for de som vil prøve seg.
La verken mennesker eller hendelser ta livsmotet fra deg.
Marie Curie, kjemiker og fysiker.

[tex]\large\dot \rho = -\frac{i}{\hbar}[H,\rho][/tex]
daofeishi
Tyrann
Tyrann
Innlegg: 1486
Registrert: 13/06-2006 02:00
Sted: Cambridge, Massachusetts, USA

Post lagt til ved et uhell. Se under.
Sist redigert av daofeishi den 17/01-2008 11:07, redigert 1 gang totalt.
daofeishi
Tyrann
Tyrann
Innlegg: 1486
Registrert: 13/06-2006 02:00
Sted: Cambridge, Massachusetts, USA

Ser nå at det eksakte svaret i lukket form kanskje er litt komplisert å komme fram til, ihvertfall for VGS-studenter. Kanskje noen finner fram til den rekursive løsningen?

Svaret jeg kom fram til nå, etter en del klabb og babb, hjelp fra mathematica og identiteter fra mathworld:

For alle heltall k>1:
[tex]\int \frac{1}{(x^2+1)^k} \rm{d}x = \frac{8 \pi (2k-3)!}{4^k (k-1)! (k-2)!}[/tex]

Ny oppgave: Bruk teknikken til å finne [tex]\int _0 ^1 x^2 e^{x} \rm{d}x[/tex]. Hint: Se på [tex]\int _0 ^1 e^{nx} \rm{d}x[/tex]
daofeishi
Tyrann
Tyrann
Innlegg: 1486
Registrert: 13/06-2006 02:00
Sted: Cambridge, Massachusetts, USA

... og ved nærmere inspeksjon, ser jeg at uttrykket over kan skrives:

[tex]\int \frac{1}{(x^2+1)^k} \rm{d}x = {2k-3 \choose k-1} \frac{8 \pi}{4^k}[/tex]
Sonki
Cayley
Cayley
Innlegg: 88
Registrert: 21/06-2007 13:31

ok, gjør et forsøk her:

som hintet så setter vi:

[tex]e^x = e^{nx}[/tex] hvor [tex]n = 1[/tex]
og
[tex]\int _0^1\ e^{nx}\rm{d}x = \frac{e^n-1}{n} [/tex]
videre får vi:
[tex]\int_0^1\ \frac{\partial}{\partial n}e^{nx}\rm{d}x = \frac{\partial}{\partial n}\frac{e^n-1}{n}[/tex]
[tex]\int_0^1\ xe^{nx}\rm{d}x = \frac{e^n(n-1)+1}{n^2}[/tex]
og dermed:
[tex]\int_0^1\ \frac{\partial}{\partial n}xe^{nx}\rm{d}x = \frac{\partial}{\partial n}\frac{e^n(n-1)+1}{n^2}[/tex]
[tex]\int_0^1\ x^2e^{nx}\rm{d}x = \frac{e^n(n^2-2n+2)-2}{n^3}[/tex]
sett [tex]n=1[/tex] og får... :)
[tex]\int_0^1\ x^2e^x\rm{d}x = \frac{e(1-2+2)-2}{1^3} = e-2[/tex]
stemmer det? :D
daofeishi
Tyrann
Tyrann
Innlegg: 1486
Registrert: 13/06-2006 02:00
Sted: Cambridge, Massachusetts, USA

Helt rett! :D
sEirik
Guru
Guru
Innlegg: 1551
Registrert: 12/06-2006 21:30
Sted: Oslo

Nå er jeg spent på om det finnes et lettfattelig bevis en plass på at metoden er lov å bruke!
daofeishi
Tyrann
Tyrann
Innlegg: 1486
Registrert: 13/06-2006 02:00
Sted: Cambridge, Massachusetts, USA

Ta en titt her

Neste oppgave:
Bruk metoden på [tex]\int _0 ^1 x^m \rm{d}x[/tex] for å finne et uttrykk for [tex]\int _0 ^1 x^m (\ln x)^n \rm{d}x[/tex] (m > -1, n positivt heltall)
Sonki
Cayley
Cayley
Innlegg: 88
Registrert: 21/06-2007 13:31

Kule saker dette :D
her er et løsningsforslag:
Begynner med å sette:
[tex]\int_0^1 x^m \rm{d}x = \int_0^1 x^{mk} \rm{d}x[/tex]
Hvor [tex]k = 1[/tex]
videre:
[tex]\int_0^1 x^{mk} \rm{d}x = \frac{x^{mk+1}}{mk+1} [/tex]
[tex]\int_0^1 \frac{\partial}{\partial k} x^{mk} \rm{d}x = \frac{\partial}{\partial k} \frac{x^{mk+1}}{mk+1}[/tex]
[tex]m\int_0^1 x^{mk} (\ln x) \rm{d}x = (-1)*\frac{m}{(mk+1)^2}[/tex]
deler med [tex]m[/tex]
[tex]\int_0^1 x^{mk} (\ln x) \rm{d}x = (-1)*\frac{1}{(mk+1)^2}[/tex]
Observerer at for videre derivasjon økes eksponenten til [tex]\ln x[/tex] med [tex]1[/tex] for hver gang, mens venstresiden vil bli multiplisert med eksponen til [tex](mk +1)[/tex] (som er negativ, noe som gir at den vil vekselsvis skifte mellom å være positiv og negativ, negativ for partall eksponen). [tex]m[/tex] vil være en felles faktor for begge, og trengs dermed ikke å tas med i beregningen. antar dermed at:
[tex]\int_0^1 x^{mk} (\ln x)^n \rm{d}x = (-1)^n*\frac{n!}{(mk+1)^{n+1}} [/tex]
beviser det med induksjon.:
Ser at det stemmer for [tex]n = 1[/tex]
antar så at det stemmer for [tex]n[/tex], og undersøker for [tex]n+1[/tex]. Undersøker altså om [tex]\int_0^1 x^{mk} (\ln x)^{n+1} \rm{d}x = (-1)^{n+1}*\frac{(n+1)!}{(mk+1)^{n+2}}[/tex] stemmer:
[tex]\int_0^1 \frac{\partial}{\partial k} x^{mk} (\ln x)^n \rm{d}x = \frac{\partial}{\partial k}(-1)^n*\frac{n!}{(mk+1)^{n+1}} [/tex]
[tex]m\int_0^1 x^{mk} (\ln x)^{n+1} \rm{d}x = (-1)^n *\frac{n!*(-(n+1))*m}{(mk+1)^{n+2}} = (-1)^n*(-1)*\frac{n!*(n+1)*m}{(mk+1)^{n+2}}[/tex]
deler med [tex]m[/tex] og ordner på uttrykkene, altså: [tex]\int_0^1 x^{mk} (\ln x)^{n+1} \rm{d}x = (-1)^{n+1}*\frac{(n+1)!}{(mk+1)^{n+2}}[/tex], som var det vi skulle bevise, antagelsen stemmer.
k = 1 gir:
[tex]\int_0^1 x^{m} (\ln x)^n \rm{d}x = (-1)^n*\frac{n!}{(m+1)^{n+1}} [/tex], som var det vi ble spurt om å finne :)
er det rett? :)
daofeishi
Tyrann
Tyrann
Innlegg: 1486
Registrert: 13/06-2006 02:00
Sted: Cambridge, Massachusetts, USA

Flott, Sonki! :)

Neste oppgave:
Bruk teknikken til aa finne et generelt uttrykk for
[tex]\int _0 ^{\frac \pi 2} x^n \sin (x) \rm{d}x[/tex]
for ikke-negative heltall n.
Svar